Please help me on this

Please Help Me On This

Answers

Answer 1

Answer19/40=x/3.6

By cross multiplication

19*3.6=40*x

68.4 =40x

x=68.4/40

x=1.71

x=

Answer 2

Answer: C

Step-by-step explanation:

[tex]\frac{19}{40} = \frac{x}{3.6}[/tex]     solve by cross product

40x = 68.4   Divide both sides by 40

x =  1.71


Related Questions

Please answer I need help!

Answers

Answer:

c & d

Step-by-step explanation:

the description matches the information in the table

Answer: A, B, C

Step-by-step explanation:

domain = x

range = y

Use the given information to find the P-value. Also, use a 0.05 significance level and state the conclusion about the null hypothesis (reject the null hypothesis or fail to reject the null hypothesis). The test statistic in a left-tailed test is z = -1.63.
a. 0.1032; fail to reject the null hypothesis
b. 0.0516; reject the null hypothesis
c. 0.9484; fail to reject the null hypothesis
d. 0.0516; fail to reject the null hypothesis

Answers

Answer:

Option d

Step-by-step explanation:

The p-value is 0.0516 which is not statistically significant to reject the null hypothesis. Thus we will fail to reject the null hypothesis.

A marketing consultant was hired to visit a random sample of five sporting goods stores across the state of California. Each store was part of a large franchise of sporting goods stores. The consultant taught the managers of each store better ways to advertise and display their goods. The net sales for 1 month before and 1 month after the consultant's visit were recorded as follows for each store (in thousands of dollars):_________.
Before visit: 57.1 94.6 49.2 77.4 43.2After visit: 63.5 101.8 57.8 81.2 41.9Do the data indicate that the average net sales improved? (Use a= 0.05)

Answers

Answer:

Step-by-step explanation:

Corresponding net sales before 1 month and after 1 month form matched pairs.

The data for the test are the differences between the net sales before and after 1 month.

μd = the​ net sales before 1 month minus the​ net sales after 1 month.

Before after diff

57.1 63.5 - 6.4

94.6 101.8 - 7.2

49.2 57.8 - 8.6

77.4 81.2 - 3.8

43.2 41.9 1.3

Sample mean, xd

= (- 6.4 - 7.2 - 8.6 - 3.8 + 1.3)/5 = - 4.94

xd = - 4.94

Standard deviation = √(summation(x - mean)²/n

n = 5

Summation(x - mean)² = (- 6.4 + 4.94)^2 + (- 7.2 + 4.94)^2 + (- 8.6 + 4.94)^2+ (- 3.8 + 4.94)^2 + (1.3 + 4.94)^2 = 60.872

Standard deviation = √(60.872/5

sd = 3.49

For the null hypothesis

H0: μd ≥ 0

For the alternative hypothesis

H1: μd < 0

The distribution is a students t. Therefore, degree of freedom, df = n - 1 = 5 - 1 = 4

2) The formula for determining the test statistic is

t = (xd - μd)/(sd/√n)

t = (- 4.94 - 0)/(3.49/√5)

t = - 3.17

We would determine the probability value by using the t test calculator.

p = 0.017

Since alpha, 0.05 > than the p value, 0.017, then we would reject the null hypothesis. Therefore, at 5% significance level, the data indicate that the average net sales improved.

The company produces two types of goods in quantities of x and y, with market prices of €40 and 80€, respectively. If the production cost is given by function C(x,y) =2x^2+5y^2+120 and is not exceeding €250. What is the max profit obtained?

Answers

Answer:

€ 270

Step-by-step explanation:

Since the production cost C(x,y) = 2x² + 5y² + 120 is less than or equal to 250, we have  2x² + 5y² + 120 ≤ 250

The selling price S(x,y) = 40x + 80y

The profit P(x,y) = S(x,y) - C(x,y) = 40x + 80y - 2x² - 5y² - 120

Using the principle of lagrange multipliers, we want to maximize the profit P(x,y) under the condition that C(x.y) ≤ 250.

So, dP/dx = 40 - 4x , dC/dx = 4x, dP/dy = 80 - 10y , dC/dy = 10y

dP/dx + λdC/dx = 0

40 - 4x + 4λx = 0  (1)

4λx = 4x - 40

λ = (x - 10)/x

dP/dy + λdC/dy = 0

80 - 10y + 10λy = 0 (2)

substituting λ into (2), we have

80 - 10y + 10(x - 10)y/x = 0

multiplying through by x, we have

80x - 10xy + 10xy - 100y = 0

80x - 100y = 0

80x = 100y

x = 100y/80

x = 5y/4

substituting x into C(x,y) ≤ 250, we have

2(5y/4)² + 5y² + 120 ≤ 250

25y²/8 + 5y² + 120 ≤ 250

25y² + 40y² + 960 ≤ 2000

65y² ≤ 2000 - 960

65y² ≤ 1040

y² ≤ 1040/65

y² ≤ 16

y ≤ ±√16

y ≤ ± 4 since its quantity, we take the positive value.

So x = 5y/4 = 5(± 4)/4 = ± 5

So, x ≤ ± 5

For the maximum value for the profit, P(x,y), we take the maximum values of x and y which are x = 5 and y = 4. Substituting these values into P(x,y), we have

P(5,4) = 40(5) + 80(4) - 2(5)² - 5(4)² - 120

= 200 + 320 - 50 - 80 - 120

= 520 - 250

= 270

So, the maximum profit obtained is € 270

A random sample of math majors taking an introductory statistics course were surveyed after completing the final exam. They were asked, "How many times did you review your final exam before handing it in to the professor?" The results are displayed in a probability density function for the random variable X, the number of times students reviewed their exam before handing it in. Find the standard deviation of X. Round the final answer to two decimal places. x P(X = x) 1 1/5 2 2/5 7 2/5

Answers

Answer:

[tex] E(X) =1 *\frac{1}{5} +2 *\frac{2}{5} +7*\frac{2}{5}= 3.8[/tex]

Now we can find the second moment with this formula:

[tex] E(X^2) = \sum_{i=1}^n X^2_i P(X_i)[/tex]

And replacing we got:

[tex] E(X^2) =1^2 *\frac{1}{5} +2^2 *\frac{2}{5} +7^2*\frac{2}{5}= 21.4[/tex]

The variance would be given by:

[tex] Var(X) =E(X^2) -[E(X)]^2 = 21.4 -[3.8]^2 = 6.96[/tex]

And the deviation would be:

[tex] Sd(X) =\sqrt{6.96}= 2.638[/tex]

Step-by-step explanation:

For this case we have the following distribution given:

X      1      2      7

P(X) 1/5  2/5   2/5

We need to begin finding the mean with this formula:

[tex] E(X) = \sum_{i=1}^n X_i P(X_i)[/tex]

And replacing we got:

[tex] E(X) =1 *\frac{1}{5} +2 *\frac{2}{5} +7*\frac{2}{5}= 3.8[/tex]

Now we can find the second moment with this formula:

[tex] E(X^2) = \sum_{i=1}^n X^2_i P(X_i)[/tex]

And replacing we got:

[tex] E(X^2) =1^2 *\frac{1}{5} +2^2 *\frac{2}{5} +7^2*\frac{2}{5}= 21.4[/tex]

The variance would be given by:

[tex] Var(X) =E(X^2) -[E(X)]^2 = 21.4 -[3.8]^2 = 6.96[/tex]

And the deviation would be:

[tex] Sd(X) =\sqrt{6.96}= 2.638[/tex]

The sales tax in Pennsylvania is 4%. If the tax on an item is $94, find the cost of the item

Answers

Answer:

$2350

Step-by-step explanation:

If 94 is 4%, multiply it by 25 to get $2350, or 100%

the cost of the item is $ 2350.

To find the cost of the item, we can set up an equation using the information given.

Let's denote the cost of the item as $ x.

According to the given information, the sales tax on the item is 4 % and is equal to $ 94. We can express this as:

0.04 x = 94

To solve for x, we divide both sides of the equation by 0.04:

x = tax on item / sales tax

x = 94 / 0.04

x = 2350

Therefore, the cost of the item is $ 2350.

Learn more about Sales Tax here

https://brainly.com/question/27608389

#SPJ2

3. The difference between two numbers is 5​

Answers

Answer:

The difference of two numbers is 5 and the difference of their reciprocals is 1/10. find the no.s

Step-by-step explanation:

⇒ x(x-5) = 50

⇒ x2 - 5x - 50 = 0  

⇒  x2 - 10x + 5x - 50 = 0  

⇒ x (x - 10) + 5 (x - 10) = 0

⇒  (x+5) (x-10) = 0

⇒   (x+5) (x-10) = 0  

⇒ x =  -5 or 10  

⇒ x = 10 (x = -5 , rejected)

Find tan x if sec x = sort 37/6 and sin x <0

Answers

Answer:

  tan(x) = -1/6

Step-by-step explanation:

We can use the relation between tan and sec:

  [tex]\displaystyle\tan{x}=\pm\sqrt{\sec^2{x}-1}\\\\\tan{x}=-\sqrt{\left(\dfrac{\sqrt{37}}{6}\right)^2-1}\quad\text{negative because sine is negative}\\\\=-\sqrt{\dfrac{37-36}{36}}=\boxed{-\dfrac{1}{6}}[/tex]

The tangent of x is -1/6.

One angle of a right triangle measures 51 degrees. What is the measure of the other small angle?

Answers

Answer:

39 degrees

Step-by-step explanation:

Given

triangle is right angled i.e one angle is 90 degrees

other angle is 51 degrees.

let the third angle be x degrees

we know that sum of angles of any triangle is 180 degrees

thus,

90 + 51+ x = 180

=> 141 + x = 180

=> x = 180 - 141 = 39.

Thus, measure of other small angle is 39 degrees.

Answer:

Step-by-step explanation:

m∠A+m∠B+m∠C = 180

90+51+x1= 180

41+x=180

x=39

In 1970, 59% of college freshmen thought that capital punishment should be abolished; by 2005, the percentage had dropped to 35%. Is the difference real, or can it be explained by chance

Answers

Answer:

there is significant distinction in opinion regarding abolition of capital punishment.

Step-by-step explanation:

Compute the p cost of 2-proportion for estimating difference. The Minitab output pronounces the p valu eto be 0.000. This is less than the assumed importance degree of alpha = 0.05. Therefore, reject null hypothesis to finish that there is significant distinction in opinion regarding abolition of capital punishment.

in the formula C=5/9(F-32),If C=35, then F=?​

Answers

Step-by-step explanation:

Hope this helps

Hope this is correct

Answer:

F = 95°

Step-by-step explanation:

[tex]C=\frac{5}{9}(F-32)[/tex] is the formula to convert Fahrenheit to Celsius

If we have C = 35, we just need to plug in this number to its corresponding variable and then solve for F

[tex]35=\frac{5}{9}(F-32)[/tex] then we need to multiply both sides of the equation by 9 to get rid of the fraction on the right side[tex]35(9)=[\frac{5}{9}(F-32)](9)[/tex] then simplifies to [tex]315=(5)(F-32)[/tex] Now we can distribute the 5 on the right side to the (F - 32) to get 315 = 5F - 160Adding 160 to both sides we get 475 = 5FDividing both sides by 5 we get 95 = F

g(x)4x^2-16x+7 completing the square

Answers

Exact form: x= plus or minus the square root of 11 plus 2

Decimal form:
X= 5.32 X= -1.32

If that’s not what your looking for then the answer is: y= 4(x-2)^2 -9

By completing the square the function will be, g(x)=4(x-2)²-9

What is standard form of the equation?

The standard form of the quadratic equation will be ax²+bx+c=0.

Equate the given equation with standard form of equation and determine the values of a, b, and c.

a=4

b=-16

c=7

For completing the square, add and subtract [tex]\frac{b^2}{4a}=\frac{(-16)^2}{4\times4}=16[/tex] in the given equation.

g(x)=4x²-16x+16-16+7

g(x)=(4x²-16x+16)-9

g(x)=4(x²-4x+4)-9

The term x²-4x+4 is equivalent to (x-2)².

g(x)=4(x-2)²-9

So, the given function is same as g(x)=4(x-2)²-9.

To learn more about the standard form of equation click:

https://brainly.com/question/29011747

#SPJ2

What are two possible measures of the angle below?

Answers

The smaller angle, inside the bold lines, is -90 degrees.

The larger angle, outside the bold lines, is 270 degrees.

Angles can be measured in increments between -90° and 630°.

What angles are created when two lines cross one other?

Two straight lines are considered to be intersecting if they come together at the same point. The intersection of two lines is known as the junction point. When two lines intersect, four angles are produced. The sum of the four angles is always 360 degrees.

Two straight lines that cross one other and produce right angles are called perpendicular lines. There are four right angles created when two perpendicular lines cross.

There are two types of angle connections produced when lines intersect:

Congruent opposite angles

Nearby angles are helpful

The information is

Let O (0, 0) be the origin, where the y and x axes must connect.

Thus, the angles on the four quadrants of the axis are produced.

The fourth quadrant's crossing lines create an angle that is given by For, the anticlockwise measure, A = -90°.

B = 360n - 180° for the clockwise measure, and n = 3 in this case.

Hence, when we simplify, we obtain

The second angle has a length of = 630°.

As a result, the angles are 90° and 630°.

Go here to find out more about angles created by intersecting lines.

https://brainly.com/question/16315358

#SPJ6

Could someone help me with this trigonometry question where you have find the x which is the adjacent. the reference angle is 39 degree and the opposide side is 30 cm.

Answers

Answer:

37.047

Step-by-step explanation:

Sin(39) = 30/hyp

Cos(39) = x/hyp

hyp = 30/Sin(39)

and hyp = x/Cos(39)

hyp = hyp

30/Sin(39) = x/Cos(39)

x = 30(Cos(39))/Sin(39)

x is approximately equal to 37.047

Please help me HURRY!!!!!!

Answers

K=-2 because the other answers are greater than -2 and the answer has to be less than or equal to

A children's roller coaster is limited to riders whose height is at least 30 inches and at most 48 inches. Write two inequalities that represent the height h of riders for the roller coaster.

Answers

Answer:

h≤48   h≥30

Step-by-step explanation:

WILL GIVE BRAINLIEST IF ANSWERED NOW
Write an equation of a line that passes through the point (3, 2) and is parallel to the line y = 3x +7
y = 3x -7
y = 1/3x+2
y= 1/3x-2

Answers

Answer:

y=3x-7

Step-by-step explanation:

lines are parallel hence gradient from the equation in question is the same as the gradient of the equation to be found.. comparing to y=mx+c, eq in question has grad 3... from the formula y-y1=m(x-x1) where (x1,y1) is equal to the point in question

Answer:

Make That Guy Brainliest Now ^^^^^    You can now that there are two answers!

Arsha predicted that she would sell 225 magnets. She actually sold 240 magnets. What are the values of a and b in the table below? Percent Error Item Approximate value Exact value Error Absolute error Ratio Percent error Magnets 225 240 a b a = Negative StartFraction 15 over 225 EndFraction; b = negative 6.7 percent a = Negative StartFraction 15 over 240 EndFraction; b = negative 6.25 percent a = StartFraction 15 over 240 EndFraction; b = 6.25 percent a = StartFraction 15 over 225 EndFraction; b = 6.7 percent

Answers

Answer:cStep-by-step Explanation:PLZS MARK BRAINLIEST PLZS PLZSMARK BRAINLIEST

Answer:

c

Step-by-step explanation:

Q 2.20: In a survey, there are two categories of respondents, employed and unemployed people, and two options, A and B. The proportion of those who have chosen option B is greater than 0.5 among the total number of the respondents, but is lower than 0.5 among the unemployed respondents. We know that 314 employed and 512 unemployed people chose option A and 356 employed chose option B. How many unemployed people chose option B

Answers

Answer:

The answer is 508

Step-by-step explanation:

Solution

First of all, the proportion of B is exceeds 0.5 in total.

Now,

To find the total of A it we have A =314 +512 = 826

The number of employed that choose B = 356

For us to have the proportion of B to be higher than the 0.5, the unemployed B from what is shown here should exceed the difference between total A and B employed

what this suggest is that the employed B is greater than 826-356 = 470

So,

The respondent that are unemployed that choose B  must be greater than 470

Thus,

We recall that the B proportion among the unemployed respondent is lesser than .50

Thus suggests that the respondent that are unemployed who choose be is lesser than 512

The conditions becomes

470 lesser than the number of unemployed respondents who selected B lesser than 512

Hence  the needed number of the number of unemployed respondents who chose B should be between 470 and 512

So, possible answer here is 508.

The lines shown below are parallel if the green line has a slope of 8 what is the slope of the redline?

Answers

Answer:

Option D

Step-by-step explanation:

If these lines are parallel, they should have the same slope. How so? Well slope is the change in axis, y / x more specifically. If the lines are parallel they should change at a similar rate so that they don't intersect, and hence are, by definition, ║;

[tex]Green Line's Slope = Red Line's Slope,\\8 = Red Line's Slope,\\Red Line's Slope = 8 units\\\\Solution - Option D[/tex]

Hope that helps!

A random sample of 150 mortgages in the state of Florida was randomly selected. From this sample, 17 were found to be delinquent on their current payment. The 98% confidence interval for the proportion based on this sample is ________.

Answers

Answer:

The 98% confidence interval for the proportion based on this sample is (0.0531, 0.1735).

Step-by-step explanation:

In a sample with a number n of people surveyed with a probability of a success of [tex]\pi[/tex], and a confidence level of [tex]1-\alpha[/tex], we have the following confidence interval of proportions.

[tex]\pi \pm z\sqrt{\frac{\pi(1-\pi)}{n}}[/tex]

In which

z is the zscore that has a pvalue of [tex]1 - \frac{\alpha}{2}[/tex].

For this problem, we have that:

[tex]n = 150, \pi = \frac{17}{150} = 0.1133[/tex]

98% confidence level

So [tex]\alpha = 0.02[/tex], z is the value of Z that has a pvalue of [tex]1 - \frac{0.02}{2} = 0.99[/tex], so [tex]Z = 2.327[/tex].

The lower limit of this interval is:

[tex]\pi - z\sqrt{\frac{\pi(1-\pi)}{n}} = 0.1133 - 2.327\sqrt{\frac{0.1133*0.8867}{150}} = 0.0531[/tex]

The upper limit of this interval is:

[tex]\pi + z\sqrt{\frac{\pi(1-\pi)}{n}} = 0.1133 + 2.327\sqrt{\frac{0.1133*0.8867}{150}} = 0.1735[/tex]

The 98% confidence interval for the proportion based on this sample is (0.0531, 0.1735).

Ronald needs a morning breakfast drink that will give him at least 390 calories. Orange juice has 130 calories in 8oz. How many ounces does he need to drink to reach his calorie goal?

Answers

Answer:

24 ounces of orange juice

Step-by-step explanation:

Given-

          Calories needed=390 calories

    Calories in 8oz juice=130 calorie

Therefore ounces of juice=(390/130)8

                                          =3 x 8

                                          =24 ounces

If Ronald needs a morning breakfast drink that will give him atleast 390 calories. Orange juice has 130 calories in 8oz. Then 24 ounces does he need to drink to reach his calorie goal.

What is Equation?

Two or more expressions with an Equal sign is called as Equation.

Ronald needs a morning breakfast drink that will give him at least 390 calories.

Orange juice has 130 calories in 8oz.

We need to find how many ounces does he need to drink to reach his calorie goal

Calories needed=390 calories

Calories in 8oz juice=130 calorie

Three hundred ninety divided by one hundred thirty times of eight.

Therefore ounces of juice=(390/130)8

Three hundred ninety divided by one hundred thirty is three.

=3 x 8

Three times of eight is twenty four.

=24 ounces

Hence 24 ounces of orange juice does he need to drink to reach his calorie goal.

To learn more on Equation:

https://brainly.com/question/10413253

#SPJ2

Records on a fleet of trucks reveal that the average life of a set of spark plugs is normally distributed with a mean of 22,100 miles. The fleet owner purchased 18 sets and found that the sample average life was 23,400 miles; the sample standard deviation was 1,412 miles.

a) To decide if the sample data support the company records that the spark plugs average 22,100 miles, state your decision in terms of the null hypothesis. Use a 0.05 level of significance.

b) What is the critical value for the test using a 0.05 level of significance?

c) What is the test statistic?

d) What is your decision?

Answers

Answer:

a) We want to conduct a hypothesis in order to see if the true mean is 22100 or not, the system of hypothesis would be:  

Null hypothesis:[tex]\mu = 22100[/tex]  

Alternative hypothesis:[tex]\mu \neq 22100[/tex]  

b) We need to find the degrees of freedom given by:

[tex] df =n-1 = 18-1=17[/tex]

And the critical values for this case are:

[tex] t_{\alpha/2}= 2.110[/tex]

c) [tex]t=\frac{23400-22100}{\frac{1412}{\sqrt{18}}}=3.906[/tex]  

d) Since the calculated value is higher than the critical value we have enough evidence to reject the null hypothesis and we can conclude that the true mean is significantly different from 221100 mi

Step-by-step explanation:

Information provided

[tex]\bar X=23400[/tex] represent the sample mean

[tex]s=1412[/tex] represent the sample standard deviation

[tex]n=18[/tex] sample size  

[tex]\mu_o =22100[/tex] represent the value to verify

[tex]\alpha=0.05[/tex] represent the significance level

t would represent the statistic (variable of interest)  

[tex]p_v[/tex] represent the p value

Part a

We want to conduct a hypothesis in order to see if the true mean is 22100 or not, the system of hypothesis would be:  

Null hypothesis:[tex]\mu = 22100[/tex]  

Alternative hypothesis:[tex]\mu \neq 22100[/tex]  

Part b

We need to find the degrees of freedom given by:

[tex] df =n-1 = 18-1=17[/tex]

And the critical values for this case are:

[tex] t_{\alpha/2}= 2.110[/tex]

Part c

The statistic is given by:

[tex]t=\frac{\bar X-\mu_o}{\frac{s}{\sqrt{n}}}[/tex]  (1)  

Replacing the info we got:

[tex]t=\frac{23400-22100}{\frac{1412}{\sqrt{18}}}=3.906[/tex]    

Part d

Since the calculated value is higher than the critical value we have enough evidence to reject the null hypothesis and we can conclude that the true mean is significantly different from 221100 mi

A certain manufactured product is supposed to contain 23% potassium by weight. A sample of 10 specimens of this product had an average percentage of 23.2 with a standard deviation of 0.2. If the mean percentage is found to differ from 23, the manufacturing process will be recalibrated.
a. State the appropriate null and alternate hypotheses.
b. Should the process be recalibrated? Explain.
c. Compute the P-value.

Answers

Answer:

(a) Null Hypothesis, [tex]H_0[/tex] : [tex]\mu[/tex] = 23%    

    Alternate Hypothesis, [tex]H_A[/tex] : [tex]\mu[/tex] [tex]\neq[/tex] 23%

(b) We conclude that the mean percentage is  different from 23 and the manufacturing process will be re-calibrated.

(c) P-value is 0.6%.

Step-by-step explanation:

We are given that a certain manufactured product is supposed to contain 23% potassium by weight.

A sample of 10 specimens of this product had an average percentage of 23.2 with a standard deviation of 0.2.

Let [tex]\mu[/tex] = mean percentage of potassium by weight.

(a) Null Hypothesis, [tex]H_0[/tex] : [tex]\mu[/tex] = 23%      {means that the mean percentage is equal to 23 and the manufacturing process will not be re-calibrated}

Alternate Hypothesis, [tex]H_A[/tex] : [tex]\mu[/tex] [tex]\neq[/tex] 23%     {means that the mean percentage is  different from 23 and the manufacturing process will be re-calibrated}

The test statistics that would be used here One-sample t-test statistics as we don't know about population standard deviation;

                                T.S. =  [tex]\frac{\bar X-\mu}{\frac{s}{\sqrt{n} } }[/tex]  ~ [tex]t_n_-_1[/tex]

where, [tex]\bar X[/tex] = sample mean percentage = 23.2

            s = sample standard deviation = 0.2

            n = sample of specimens = 10

So, the test statistics  =  [tex]\frac{23.2-23}{\frac{0.2}{\sqrt{10} } }[/tex]  ~ [tex]t_9[/tex]

                                     =  3.162

The value of t test statistic is 3.162.

Since, in the question we are not given with the level of significance so we assume it to be 5%. Now, at 5% significance level the t table gives critical value of -2.262 and 2.262 at 9 degree of freedom for two-tailed test.

(b) Since our test statistic doesn't lie within the range of critical values of t, so we have sufficient evidence to reject our null hypothesis as it will fall in the rejection region due to which we reject our null hypothesis.

Therefore, we conclude that the mean percentage is  different from 23 and the manufacturing process will be re-calibrated.

(c) The P-value of the test statistics is given by;

            P-value = P( [tex]t_9[/tex] > 3.162) = 0.006 or 0.6%

(a) Null Hypothesis,  [tex]H_o:\mu[/tex]: = 23%    

   Alternate Hypothesis, [tex]H_A:\mu\neq[/tex] : 23%

(b) We conclude that the mean percentage is  different from 23 and the manufacturing process will be re-calibrated.

(c) P-value is 0.6%.

What is a null hypothesis?

The hypothesis that there is no significant difference between specified populations, any observed difference being due to sampling or experimental error.

We are given that a certain manufactured product is supposed to contain 23% potassium by weight.

A sample of 10 specimens of this product had an average percentage of 23.2 with a standard deviation of 0.2.

Let  = mean percentage of potassium by weight.

(a) Null Hypothesis, [tex]H_o:\mu[/tex]:  = 23%      {means that the mean percentage is equal to 23 and the manufacturing process will not be re-calibrated}

Alternate Hypothesis,  [tex]H_A:\mu\neq[/tex]:   23%     {means that the mean percentage is  different from 23 and the manufacturing process will be re-calibrated}

The test statistics that would be used here One-sample t-test statistics as we don't know about population standard deviation;

[tex]TS=\dfrac{X-\mu}{\frac{s}{\sqrt{n}}}[/tex]   ~ [tex]t_{n-1}[/tex]

where,  = sample mean percentage = 23.2

           s = sample standard deviation = 0.2

           n = sample of specimens = 10

So, the test statistics  =   [tex]\dfrac{23.2-23}{\frac{0.2}{\sqrt{10}}}[/tex] ~ [tex]t_g[/tex]

                                    =  3.162

The value of t test statistic is 3.162.

Since, in the question we are not given with the level of significance so we assume it to be 5%. Now, at 5% significance level the t table gives critical value of -2.262 and 2.262 at 9 degree of freedom for two-tailed test.

(b) Since our test statistic doesn't lie within the range of critical values of t, so we have sufficient evidence to reject our null hypothesis as it will fall in the rejection region due to which we reject our null hypothesis.

Therefore, we conclude that the mean percentage is  different from 23 and the manufacturing process will be re-calibrated.

(c) The P-value of the test statistics is given by;

           P-value = P( [tex]t_g[/tex] > 3.162) = 0.006 or 0.6%

Hence ,

(a) Null Hypothesis,  [tex]H_o:\mu[/tex]: = 23%    

   Alternate Hypothesis, [tex]H_A:\mu\neq[/tex] : 23%

(b) We conclude that the mean percentage is  different from 23 and the manufacturing process will be re-calibrated.

(c) P-value is 0.6%.

To know more about null hypothesis follow

https://brainly.com/question/15980493

the ratio of the ages of Kissi and Esinam is 3:5 and that of Esinam and Lariba is 3:5 and
the sum of the ages of all 3 is 147 years, what is the age difference between oldest the youngest

Answers

Answer:

The age difference between oldest the youngest is of 48 years.

Step-by-step explanation:

We can solve this question using a system of equations.

I am going to say that:

Kissi's age is x.

Esinam's age is y.

Lariba's age is z.

The ratio of the ages of Kissi and Esinam is 3:5

This means that [tex]\frac{x}{y} = \frac{3}{5}[/tex], so [tex]5x = 3y[/tex]

That of Esinam and Lariba is 3:5

This means that [tex]\frac{y}{z} = \frac{3}{5}[/tex], so[tex]5y = 3z[/tex]

The sum of the ages of all 3 is 147 years

This means that [tex]x + y + z = 147[/tex]

What is the age difference between oldest the youngest

z is the oldest

x is the youngest.

First i will find y.

We have that, from the equations above: [tex]x = \frac{3y}{5}[/tex] and [tex]z = \frac{5y}{3}[/tex]

So

[tex]x + y + z = 147[/tex]

[tex]\frac{3y}{5} + y + \frac{5y}{3} = 147[/tex]

The lesser common multiple between 5 and 3 is 15. So

[tex]\frac{3*3y + 15*y + 5*5y}{15} = 147[/tex]

[tex]49y = 147*15[/tex]

[tex]y = \frac{147*15}{49}[/tex]

[tex]y = 45[/tex]

Youngest:

[tex]x = \frac{3y}{5} = \frac{3*45}{5} = 27[/tex]

Oldest:

[tex]z = \frac{5y}{3} = \frac{5*45}{3} = 75[/tex]

Difference:

75 - 27 = 48

The age difference between oldest the youngest is of 48 years.

Please answer this correctly

Answers

Answer:

416

Step-by-step explanation:

plz mark brainliest!

Answer:

385

Step-by-step explanation:

use l x w

14x19

16x3

7x10

385

When Ryan was born, he weighed 7 pounds.At 6 months, he weighed 11.2 pounds. Amanda weighed 6 pounds when she was born, and 12.9 pounds at 6 months. Which baby had a greater percent increase in weight? Explain

Answers

Answer:

✅Amanda had a greater percent increase in weight.

Step-by-step explanation:

The percent change in Ryan’s weight was 42/7 or 60%. The percent change in Amanda’s weight was 6.9/6, or 115%. Amanda had a greater percent increase in weight.

IamSugarBee

Answer:

The percent change in Ryan’s weight was 4.2/7, or 60%. The percent change in Amanda’s weight was 6.9/6 , or 115%. Amanda had a greater percent increase in weight.

Step-by-step explanation:

its the sample answer i just did it

Four men are to divide K500 equally among them. When the money was given, 20% was taken away.
How much each did the four men receive?​

Answers

Answer: 20% of 500= 100

So 500-100 = 400

4x100= 400

Step-by-step explanation:

The following stem-and-leaf plots compare the ages of 30 actors and 30 actresses at the time they won the Oscar award for Best Actor or Actress. Actors Stems Actresses 2 146667 98753221 3 00113344455778 88776543322100 4 11129 6651 5 210 6 011 6 7 4 8 0 (a) What is the age of the youngest actor to win an Oscar? years (b) What is the age difference between the oldest and the youngest actress to win an Oscar? years (c) What is the oldest age shared by two actors to win an Oscar?

Answers

The image of the stem-and-leaf plots is in the attachment.

Answer: (a) 31 years; (b) 59 years; (c) 56 years

Step-by-step explanation: Steam and leaf is a table that shows the digits of the data value split into a "stem", which represents the first digit, and a "leaf", which is the last digit.

For example, the first row of the table in the attachment, indicate a "stem" 2 and the first number of a "leaf" is 1, so the actress has 21 years.

(a) According to the table, the youngest actor to win an Oscar has a "stem" 3 and the first "leaf" from the right is 1, so the actor has 31 years.

(b) The oldest actress is 80 and the youngest is 21, so difference is:

80 - 21 = 59

The difference is 59 years.

(c) The oldest age shared by 2 actors is 56 years.

A supervisor records the repair cost for 14 randomly selected refrigerators. A sample mean of $79.20 and standard deviation of $10.41 are subsequently computed. Determine the 90% confidence interval for the mean repair cost for the refrigerators. Assume the population is approximately normal. Step 1 of 2 : Find the critical value that should be used in constructing the confidence interval. Round your answer to three decimal places.

Answers

Answer:

( $74.623, $83.777)

The 90% confidence interval is = ( $74.623, $83.777)

Critical value at 90% confidence = 1.645

Step-by-step explanation:

Confidence interval can be defined as a range of values so defined that there is a specified probability that the value of a parameter lies within it.

The confidence interval of a statistical data can be written as.

x+/-zr/√n

Given that;

Mean x = $79.20

Standard deviation r = $10.41

Number of samples n = 14

Confidence interval = 90%

Using the z table;

The critical value that should be used in constructing the confidence interval.

z(α=0.05) = 1.645

Critical value at 90% confidence z = 1.645

Substituting the values we have;

$79.20+/-1.645($10.42/√14)

$79.20+/-1.645($2.782189528308)

$79.20+/-$4.576701774067

$79.20+/-$4.577

( $74.623, $83.777)

The 90% confidence interval is = ( $74.623, $83.777)

Other Questions
Pls help me. I need this done ASAP thank you :) you and a group of friends spent at least $73.00 at a local pizzeria. Drinks for the table totaled $13 and it was $15 per pizza. how many pizzas could they order what was the decision in plessy v. ferguson? What is Trichotillomania? What type of disease is it? What causes it? Option A costs an initial $2 billion and will involve variable costs (labor and material) of $5 per bottle of spirits. Option B costs an initial $4 billion and will involve variable costs (labor and material) of $3 per bottle of spirits. Assuming an annual capital charge equal to 10 percent of the initial costs, what is the average fixed cost at production level of 20,000,000 bottles per year for the Option B facility Based on your prior reading and the cartoon above, what is the "crack problem" Gorbachev faced in the late 1980s? Why did he face this problem? (Pics provided) Which substances are products of a fermentation reaction?1.water and carbon dioxide2.alcohol and carbon dioxide3.soap and glycerol4.ester and water How does the first paragraph of The Dark Game best support the central idea that the Civil War was a long war?(AlbeO It shares the detail that the war lasted four years.O It states what people believed about the war at the time.O It tells how many soldiers actually died in the war.O. It gives an example of a battle that lasted almost two days. Which story premise is most clearly a classic tragedy?O A. Nathaniel spends a long time building his house, but it still fallsapart.B. Over the course of Mia's life, one unlucky thing after anotherhappens.O C. Hurricane Don approaches land, forcing Drew to leave his homebehind.D. Gillian starts a successful company, but her arrogance brings itdown.Please help me Select the correct word to complete each sentence.When writing a letter to your grandma from Arkansas, you would use English.Some business leaders use when they talk about "performance results" or "branding". what is the purpose of the print area feature The Wild Dogs and the Rattlers also have membership programs for their fans. The Wild Dogs program has an initial fee of $20 and a discounted single-game ticket price of $1.50. The Rattlers program has an initial fee of $14 and a discounted ticket price of $2.25 per game. Which linear equation represents the Wild Dogs program? Let x represent the number of games and y the total c 10. Each year, the government establishes an income cap forMedicare taxesunemployment taxesindividual income taxesSocial Security taxes How did the Destroyers for Bases agreement President Roosevelt signed help Britain and its allies?1. The agreement put US bases on Soviet territory.2. The agreement put US bases on British territory. 3.The agreement put US destroyers in Soviet waters.4. The agreement put US destroyers in German waters. The formula for the area of a triangle is , where b is the length of the base and h is the height.Find the height of a triangle that has an area of 30 square units and a base measuring 12 units.3 units What can readers conclude about Allison? Check all that apply Water, in a 100-mm-diameter jet with speed of 30 m/s to the right, is deflected by a cone that moves to the left at 14 m/s. Determine (a) the thickness of the jet sheet at a radius of 230 mm. and (b) the external horizontal force needed to m 7A CD holds 650 MB of data. A DVD holds 4 GB of data. How many CDs would you needto equal one DVD In Cruel Tribute, which characters actions most advance the development of the plot? Ariadne King Minos the Minotaur Theseus What steps should be taken to complete the conversion? StartFraction 8 miles Over 1 EndFraction times StartFraction 1.61 kilometers Over 1 mile EndFraction times StartFraction 1000 meters Over 1 kilometer EndFraction = blank